Đến nội dung

mrjackass nội dung

Có 114 mục bởi mrjackass (Tìm giới hạn từ 09-06-2020)



Sắp theo                Sắp xếp  

#378695 Tìm min $\sum \frac{x^{3}}{y^{2...

Đã gửi bởi mrjackass on 18-12-2012 - 22:00 trong Bất đẳng thức và cực trị

Cho biết x,y,z thực dương và $x^{2}+y^{2}+z^{2}\geq 1$
Tìm GTNN biểu thức $\sum \frac{x^{3}}{y^{2}+z^{2}}$

Công thức kẹp trong cặp dấu $
$công thức$



#378714 Cho tứ giác $ABCD$ nội tiếp đường tròn tâm O bán kính R. CMR nếu...

Đã gửi bởi mrjackass on 18-12-2012 - 22:31 trong Hình học

Mình xin được đề xuất 1 cách giải khác
Trc hết, ta có bổ đề: $\Delta ABC$ nội tiếp đường tròn (O;R) thì a=sinA.2R (phần CM khá dễ nên dành cho các bạn)
Áp dụng vào bài toán trên: Đặt $\widehat{ACB}$=x, $\widehat{CBD}$=y. Theo bổ đề ta có:
$AB^{2}+CD^{2}=4R^{2}.sin^{2}x+4R^{2}.sin^{2}y=4R^{2}.(sin^{2}x+sin^{2}y)=4R^{2}$
=> $sin^{2}x+sin^{2}y=1$ => $sin y = cos x$ => $widehat{ACB}+widehat{CBD}=90$ => đpcm



#379088 CMR: $\sum \frac{a_1^{m}}{S(p)-a_1^...

Đã gửi bởi mrjackass on 20-12-2012 - 16:54 trong Bất đẳng thức và cực trị

Cho $a_1,a_2,...,a_n>0$ và m,p nguyên dương.
Kí hiệu $S(i)=a_1^{i}+a_2^{i}+...+a_n^{i}$
CMR: $\sum \frac{a_1^{m}}{S(p)-a_1^{p}}\geq \frac{n.[S(m)]}{(n-1).[S(p)]}$



#380192 Chứng minh: $\sum \frac{a}{(b+c)^{2}...

Đã gửi bởi mrjackass on 24-12-2012 - 22:46 trong Bất đẳng thức và cực trị

Cho a,b,c dương. CMR:
$\sum \frac{a}{(b+c)^{2}}\geq \frac{9}{4(a+b+c)}$



#381277 Dựng đường thẳng d sao cho chu vi $\Delta ADD_1$ lớn nhất

Đã gửi bởi mrjackass on 28-12-2012 - 19:23 trong Hình học

Cho $\Delta ABC$ điểm $I$ thuộc cạnh $BC$ cố định. Dựng đ.tròn ngoại tiếp $\Delta AIB$ và $\Delta AIC$ kí hiệu là $(AIB)$ và $(AIC)$. 1 đường thẳng $d$ đi qua $I$ cắt $(AIB)$ và $(AIC)$ tại $B_1$ và $C_1$. $D$ là 1 điểm cố định khác thuộc $BC$ ($D\neq I$). $D_1$ là 1 điểm thuộc BC và thỏa mãn $\frac{BD}{DC}= \frac{B_1D_1}{D_1C_1}$
Tìm cách dựng đường thẳng $d$ sao cho
a)Chu vi $\Delta ADD_1$ lớn nhất
b)Diện tích $\Delta ADD_1$ lớn nhất
MOD : Chú ý tiêu đề ! :angry:



#381303 Chứng minh rằng: M là trung điểm CD

Đã gửi bởi mrjackass on 28-12-2012 - 20:02 trong Hình học

Cho (O;R) và (O';r) cắt nhau tại A và B. Tiếp tuyến chung CD; AB cắt CD tại M. OO' cắt (O) và (O') lần lượt tại E và F. Chứng minh rằng:
a) M là trung điểm CD
b) CE, DF, AB đồng quy



#381423 CMR: tâm của đường tròn đi qua 3 điểm M,N,P luôn chạy trên đường thẳng cố định

Đã gửi bởi mrjackass on 28-12-2012 - 23:18 trong Hình học

Ta có $\angle MNO=\angle MPO=90^{\circ}$ nên tứ giác MNOP nội tiếp đường tròn đường kính OM. Gọi trung điểm của OM là I => tâm ngoại tiếp $\Delta MNP$ là I. Kẻ OH vuông góc với AB => OH=a là độ dài cố định. Kẻ IK vuông góc với d thì theo t/c đường trung bình $IK=\frac{a}{2}$ Do đó I chạy trên đường thẳng e//d và cách d 1 khoảng là a/2

Bổ sung: e thuộc nửa mp bờ AB chứa O



#381960 $\frac{a}{7a^{2}+11}+\frac{...

Đã gửi bởi mrjackass on 30-12-2012 - 13:10 trong Bất đẳng thức và cực trị

Cho $a,b,c> 0$ và $a+b+c= 3$ Chứng minh rằng:
$\frac{a}{7a^{2}+11}+\frac{b}{7b^{2}+11}+\frac{c}{7c^{2}+11}\leq \frac{1}{6}$



#381963 $\frac{a^3b}{1+ab^2}+\frac{b^3c}...

Đã gửi bởi mrjackass on 30-12-2012 - 13:19 trong Bất đẳng thức và cực trị

Cho a,b,c là các số thực dương. CMR:
$\frac{a^3b}{1+ab^2}+\frac{b^3c}{1+bc^2}+\frac{c^3a}{1+ca^2}\geq \frac{abc(a+b+c)}{1+abc}$



#382416 BĐT kết thúc 2012, chào 2013

Đã gửi bởi mrjackass on 31-12-2012 - 23:56 trong Bất đẳng thức và cực trị

Mình xim được kết thúc năm 2012 của box BĐT và cực trị bằng bài toán sau:
Cho $a_1, a_2,..., a_n$ và $b_1, b_2,..., b_n$ là 2 bộ số thực dương; $m$ là 1 số nguyên dương thỏa mãn $a_1^{m}+a_2^{m}+...+a_n^{m}=b_1^{m}+b_2^{m}+...+b_n^{m}$. Chứng minh rằng:
$\frac{a_1^{2013+m}}{b_1^{m}}+\frac{a_2^{2013+m}}{b_2^{m}}+...+\frac{a_n^{2013+m}}{b_n^{m}}\geq a_1^{2013}+a_2^{2013}+...+a_n^{2013}$



#382420 D,E,F là chân 3 đường cao

Đã gửi bởi mrjackass on 01-01-2013 - 00:11 trong Hình học

Chúc mừng năm mới 2013! Mình xin đưa ra giả thiết sau mở hàng cho box, mong các bạn giải quyết được trong thời gian sớm nhất:
Cho tam giác ABC nhọn và D,E,F lần lượt thuộc, BC, CA, AB thỏa mãn 4 tam giác ABC, AEF, DEC, DBE đồng dạng. Chứng minh rằng: D, E, F là chân 3 đường cao tam giác ABC



#383915 D,E,F là chân 3 đường cao

Đã gửi bởi mrjackass on 05-01-2013 - 19:45 trong Hình học

Không. Bạn lưu ý mình đã viết các tam giác đồng dạng theo đỉnh tương ứng. Ví dụ như tam giác ABC đồng dạng với tam giác DEC thì $\widehat{ABC}= \widehat{DEC}$. Bạn đã hiểu sai giả thiết
____________________________
Beautifulsunrise: Theo như bạn nói thì $\widehat{ABC}= \widehat{DEC}= \widehat{DBE}$



#392582 $Q=a^{2}(a+1)+b^{2}(b+1)$

Đã gửi bởi mrjackass on 02-02-2013 - 20:06 trong Bất đẳng thức và cực trị

Bài này khó nhưng thuộc dạng có phương pháp. Phương pháp của nó là Nhóm Abel.
Trước hết, ta có công thức biến đổi Abel như sau: $\sum_{i=1}^{n}a_ib_i=\sum_{i=1}^{n}(a_i-a_{i+1})(\sum_{j=1}^{i}b_j)$, trong đó $a_{n+1}=0$
Cụ thể, với n=2 thì công thức sẽ là $a_1b_1+a_2b_2=(a_1-a_2)b_1+a_2(b_1+b_2)$
Quay lại bài toán, ta dự đoán dấu "=" xảy ra khi Q đạy GTLN là tại $a=2$ và $b=3$
Như vậy ta sẽ đưa bài toán về chứng minh bất đẳng thức: $Q \leq 2^3 + 2^2 + 3^2 +3^3$ bằng cách chứng minh 2 bất đẳng thức nhỏ là $a^2+b^2 \leq 2^2 + 3^2$ (1) và $a^3+b^3 \leq 2^3 + 3^3$ (2)
Chứng minh (1): Nếu có 1 số trong a hoặc b nhỏ hơn 2 thì ta có ngay đpcm. Xét trường hợp $ 2 \leq a \leq b \leq 3$. Đổi vế và áp dụng hằng đẳng thức $a^2-b^2=(a-b)(a+b)$, ta được đpcm
<=>$(a-2)(a+2)+(b-3)(b+3) \leq 0$ (*)
Bây giờ ta tìm cách chọn $a_1$, $a_2$, $b_1$, $b_2$ cho hợp lý. Nhận thấy $(a-2)(a+2) \geq 0$ nên ta muốn bổ sung 1 đại lượng để biểu thức trên không dương, mặt khác ta cần dấu "=" khi $a=2$ và $a+b=5$ nên ta chọn $a-2$ là $b_1$, $b-3$ là $b_2$. Do đó thì $a_1$ là $a+2$; $a_2$ là $b+3$. Áp dụng công thức Abel:
(*) <=> $(a+2-b-3)(a-2)+(a+2)(a+b-5) \leq 0$
Dễ thấy bất đẳng thức trên đúng.
Tương tự, ta CM (2). Nếu a hoặc b có 1 số nhỏ hơn 2 thì có ngay đpcm. Xét trường hợp $ 2 \leq a \leq b \leq 3$ đpcm
<=> $(a-2)(a^2+2a+4)+(b-3)(b^2+3b+9)\leq 0$
<=> $(a^2+2a+4-b^2-3b-9)(a-2)+(b^3+3b+9)(a+b-5) \leq 0$
Dễ thấy bất đẳng thức trên đúng.
Tóm lại ta có $max Q =2^2+3^2+2^3+3^3$. Dấu "=" <=> $a=2$ và $b=3$



#393810 Chứng minh rằng f(x)=ax với a là hằng số khác 0.

Đã gửi bởi mrjackass on 06-02-2013 - 15:59 trong Đại số

Nếu $x_1=x_2$ thì $f(x)$ là hàm nào chẳng được :|



#394025 Tìm $GTNN$ của $A=a^3+b^3+c^3$

Đã gửi bởi mrjackass on 06-02-2013 - 20:35 trong Bất đẳng thức và cực trị

Bài này trong cuốn "Nâng cao và phát triển Toán 9 tập một" của Vũ Hữu Bình. Mình xin được viết lại lời giải của tác giả (Chứ không phải lời giải của mình)
Ta có: $a^3+1=(a+1)(a^2-a+1=(a+1)(a^2-a+\frac{1}{4})+\frac{3}{4}(a+1)$
Do đó: $a^3-\frac{3}{4}a+\frac{1}{4}=(a+1)(a-\frac{1}{2})^2\geq0$ với $a \geq -1$
Lập các đánh giá tương tự với b và c
=> $(a^3+b^3+c^3)-\frac{3}{4}(a+b+c)+\frac{3}{4}\geq 0 \iff a^3+b^3+c^3 \geq -\frac {3}{4}$
Dấu "=" khi có 1 số bằng -1 và 2 số bằng $\frac{1}{2}$



#394035 Cho a,b,c không âm thoả mãn ab+bc+ca=1. Tìm GTNN của biểu thức:

Đã gửi bởi mrjackass on 06-02-2013 - 20:53 trong Bất đẳng thức và cực trị

Phương pháp này gọi là cân bằng hệ số trong bất đẳng thức. việc tìm ra các hệ số trên cần giải một hệ các phương trình. :D Bạn thử làm xem thế nào.

Cách làm của mình như sau:
Thấy được vai trò của $b;c$ là tương đương, ta dự đoán dấu "=" xảy ra khi $\alpha a=b=c$, trong đó $\alpha$ là 1 hằng số dương mà ta sẽ đi tìm. Bây giờ ta cứ "hồn nhiên" AM-GM như sau:
$\alpha^2 a^2+b^2\geq 2\alpha ab$
$\alpha^2 a^2+c^2\geq 2\alpha ac$
$\alpha(b^2+c^2)\geq 2\alpha bc$
Cộng 3 vế các đánh giá trên: $\alpha^22a^2+(\alpha+1)(b^2+c^2)\geq2\alpha(ab+bc+ca)=2\alpha$
Ta thấy vế trái hệ số của $a^2$ và $b^2+c^2$ còn "lệch", chưa thể đưa về $2a^2+b^2+c^2$. Do đó ta sẽ "chỉnh" $ \alpha $ sao cho $ \alpha^2=\alpha+1 $
Giải phương trình trên, lấy nghiệm dương, ta được $\alpha=\frac{\sqrt{5}+1}{2}$. Lúc này thì thay vào các đánh giá trên và tìm dấu "=" thôi



#394146 Lập pt bậc hai có hệ số nguyên nhận a-1 là 1 nghiệm

Đã gửi bởi mrjackass on 06-02-2013 - 23:17 trong Đại số

$a=\sqrt{7}$ (bạn tự quy đồng).Đặt $x=a-1=\sqrt{7}-1 \iff x+1=\sqrt{7} \iff x^2+2x+1=7 \iff x^2+2x-6=0$. Đó là PT cần lập



#394273 $\left\{\begin{matrix} a^{2}+b^...

Đã gửi bởi mrjackass on 07-02-2013 - 12:05 trong Đại số

Giải hệ phương trình sau trên tập $\mathbb{R}^{+}$:

$\left\{\begin{matrix} a^{2}+b^{2}+c^{2}\leq 3\\ \frac{a}{\sqrt{b+c}}+\frac{b}{\sqrt{c+a}}+\frac{c}{\sqrt{a+b}}\leq \frac{\sqrt{2}}{2}(a+b+c) \end{matrix}\right.$

Đề kiểu này dễ đoán hướng làm quá.
Đặt $P=\frac{a}{\sqrt{b+c}}+\frac{b}{\sqrt{c+a}}+\frac{c}{\sqrt{a+b}}$
Theo C-S: $a+b+c \leq \sqrt{3(a^2+b^2+c^2)} \leq 3$
Do vai trò $a;b;c$ tương đương nên giả sử $a \geq b \geq c$. Khi đó thì $\frac{1}{\sqrt{b+c}} \geq \frac{1}{\sqrt{c+a}} \geq \frac{1}{\sqrt{a+b}}$
Áp dụng BĐT Chebyshev, AM - GM và đánh giá $a+b+c \leq 3$:
$3P \geq (a+b+c)(\frac{1}{\sqrt{b+c}}+\frac{1}{\sqrt{c+a}}+\frac{1}{\sqrt{a+b}})\geq(a+b+c)(\frac{3}{\sqrt{\sqrt[3]{(a+b)(b+c)(c+a)}}})\geq (a+b+c)(\frac{3}{\sqrt{\frac{2(a+b+c)}{3}}})\geq (a+b+c)\frac{3}{\sqrt{2}} \iff P \geq \frac{\sqrt{2}}{2}(a+b+c)$
Tuy nhiên theo hệ phương trình: $ P \leq \frac{\sqrt{2}}{2}(a+b+c)$. Do đó $ P = \frac{\sqrt{2}}{2}(a+b+c)$
Dấu bằng phải xảy ra ở các đánh giá trên để thỏa mãn, tức là $a=b=c=1$
Kết luận: Hệ có nghiệm duy nhất $(a;b;c)$ là $(1;1;1)$



#394317 $\frac{a^2+b^2}{a+b}+\frac{b^2+c^2...

Đã gửi bởi mrjackass on 07-02-2013 - 13:55 trong Bất đẳng thức và cực trị

Cho $a,b,c \geq 0$. CMR:
$\frac{a^2+b^2}{a+b}+\frac{b^2+c^2}{b+c}+\frac{c^2+a^2}{c+a} \leq\frac{3(a^2+b^2+c^2)}{a+b+c}$



#394553 CMR: Với mọi số nguyên dương n, số $a^n$ viết được dưới dạng $...

Đã gửi bởi mrjackass on 07-02-2013 - 20:50 trong Đại số

Cho $a= \sqrt{2}-1$
CMR: Với mọi số nguyên dương n, số $a^n$ viết được dưới dạng $\sqrt{m}-\sqrt{m-1}$

Nếu $n=1$ thì $a_1=\sqrt{2}-\sqrt{1}$
Giả sử, giả thiết đúng tới $n=k$, tức là $a^k=\sqrt{i}-\sqrt{i-1}$
Ta cần chứng minh $a^{k+1}$ có dạng $\sqrt{j}-\sqrt{j-1}$
Xét: $a^{k+1}=a^k.a=(\sqrt{i}-\sqrt{i-1})(\sqrt {2}-1)=\sqrt{2i}-\sqrt{2(i-1)}-\sqrt{i}+\sqrt{i-1}=(\sqrt{2i}+\sqrt{i-1})-(\sqrt{2(i-1)}+\sqrt{i})$
Đặt $\sqrt{j}=\sqrt{2i}+\sqrt{i-1} \iff j=3i-1+\sqrt{2i(i-1)} \iff j-1=3i-2+\sqrt{2i(i-1)}=2(i-1)+i+\sqrt{2(i-1)i}=(\sqrt{2(i-1)}+\sqrt{i})^2$
=>$\sqrt{2(i-1)}+\sqrt{i}=\sqrt{j-1}$ => $a^{k+1}=\sqrt{j}-\sqrt{j-1}$ => đpcm
Bài toán có thể được mở rộng như sau:
Cho $a=\sqrt{t+1}-\sqrt{t}$
Chứng minh rằng: với mọi n nguyên dương thì $a^n$ luôn viết được dưới dạng $\sqrt{x+1}-\sqrt{x}$
Cách làm tương tự

Trang ơi không có điều kiện gì của $m$ à ?

Chỉ cần căn thức có nghĩa là được thôi bạn



#394573 $\sqrt[3]{x^2}+\sqrt[3]{y^2}=\sqrt[3]...

Đã gửi bởi mrjackass on 07-02-2013 - 21:14 trong Đại số

Chứng minh rằng nếu $\sqrt{x^2+\sqrt[3]{x^4y^2}}+\sqrt{y^2+\sqrt[3]{x^2y^4}}=a$ thì $\sqrt[3]{x^2}+\sqrt[3]{y^2}=\sqrt[3]{a^2}$.

Trước hết ta xét: $(x^{2}+\sqrt[3]{x^4y^2})(y^{2}+\sqrt[3]{x^2y^4})=x^2y^2+\sqrt[3]{x^4y^8}+\sqrt[3]{x^8y^4}+x^2y^2=(\sqrt[3]{x^4y^2})^2+(\sqrt[3]{x^2y^4})^2+2\sqrt[3]{x^8y^8}=(\sqrt[3]{x^4y^2}+\sqrt[3]{x^2y^4})^2$
Ta có: $\sqrt{x^2+\sqrt[3]{x^4y^2}}+\sqrt{y^2+\sqrt[3]{x^2y^4}}=a \iff x^2+y^2+\sqrt[3]{x^4y^2}+\sqrt[3]{x^2y^4}+2\sqrt{({x^2+\sqrt[3]{x^4y^2}})(y^2+\sqrt[3]{x^2y^4})}=a^2 \iff x^2 +y^2 +\sqrt[3]{x^4y^2}+\sqrt[3]{x^2y^4}+2(\sqrt[3]{x^4y^2}+\sqrt[3]{x^2y^4})=a^2 \iff x^2+y^2+3(\sqrt[3]{x^4y^2}+\sqrt[3]{x^2y^4})=a^2 \iff (\sqrt[3]{x^2}+\sqrt[3]{y^2})^3=(\sqrt[3]{a^2})^3 \iff \sqrt[3]{x^2}+\sqrt[3]{y^2}=\sqrt[3]{a^2}$
OK bạn nhé. Lúc làm mình quên không đặt $\sqrt[3]{x^2}=m$ và $\sqrt[3]{y^2}=n$ để lời giải đẹp hơn :biggrin: Mong bạn thông cảm :D



#394584 $$ (\frac{a}{a+b})^{n}+(\fr...

Đã gửi bởi mrjackass on 07-02-2013 - 21:26 trong Bất đẳng thức và cực trị

Cho a,b,c dương, n >1 (n nguyên)
Chứng minh rằng
$$ (\frac{a}{a+b})^{n}+(\frac{b}{b+c})^{n}+(\frac{c}{c+a})^{n}\geq \frac{3}{2^{n}}$$

Bạn tham khảo tại http://diendantoanho...bckgeq-frac32k/



#394599 $$ (\frac{a}{a+b})^{n}+(\fr...

Đã gửi bởi mrjackass on 07-02-2013 - 21:45 trong Bất đẳng thức và cực trị

Bài em đã giải và bài ở topic này hoàn toàn khác nhau. Về bài ở topic này có thể dùng phương pháp dồn biến, khá dài dòng ==!
Đối với bài kia em có thể dùng Chevbyshev với 2 bộ đơn điệu, cơ mà 2 bộ trong bài toán topic này lại không hề đơn điệu tăng hoặc giảm :)
Chúc em xử được bài này !
___
NLT

Xin lỗi em sơ suất quá, nhìn nhầm 2 bài toán

nhìn thì giống nhưng ko phải Nesbit tổng quát

Nó chẳng liên quan đâu bạn ạ. Nesbit (hay Shapiro) tổng quát là lquan tới số biến, còn cái này của bạn nó còn chẳng giống Nesbit 3 số :D



#394618 $\frac{a^{2}}{(b-c)^{2}}+...

Đã gửi bởi mrjackass on 07-02-2013 - 22:10 trong Bất đẳng thức và cực trị

***Lưu ý: lời giải này không phải của mình***
Đặt $x=\frac{a}{b-c};y=\frac{b}{c-a};z\frac{c}{a-b}$
Thế thì $xy+yz+zx=\frac{ab}{(b-c)(c-a)}+\frac{bc}{(c-a)(a-b)}+\frac{ca}{(a-b)(b-c)}=\frac{ab(a-b)+bc(b-c)+ca(c-a)}{(a-b)(b-c)(c-a)}=-1$ (Bạn có thể tích chéo để CM đẳng thức trên)
Mặt khác $(x+y+z)^2 \geq 0 \iff x^2+y^2+z^2 \geq -2(xy+yz+zx)=(-2)(-1)=2$
=> đpcm



#394623 $\frac{2010x+2680}{x^{2}+1}$

Đã gửi bởi mrjackass on 07-02-2013 - 22:18 trong Đại số

Tìm giá trị nhỏ nhất và lớn nhất của biểu thức : $\frac{2010x+2680}{x^{2}+1}$

Bài này miền giá trị của hàm số cũng được mà